4. Angles A and B are supplementary. If mA = 67, find mB

Answers

Answer 1

Answer:

113 degrees

Step-by-step explanation:

Two angles are called supplementary when they add up to 180 degrees

[tex]180 = x + y \\ 180 = 67 + y \\ y = 180 - 67 \\ y = 113 \\ therefore \: mB \: = 113[/tex]


Related Questions

On Monday, a packaging company put together 450x packages of 45 cashews. On Tuesday, the company put together 175 less packages with 15x more cashews in each bag. Which of the following equations could be used to determine how many cashews were packaged on Tuesday?

Answers

Answer:

275x packages of 720 cashews

Step-by-step explanation:

450-175=275

(a+1)b

a=15 and b=45

(15 + 1) x 45 = 720

5)Express in standard form: (i)4809000 (ii)0.00849​

Answers

Answer:

I hope it helps .in case you don't understand any part ,feel free to ask.

Select the correct answer. Simplify the polynomial expression. 3x(-2x+7) -5(x-1)(4x-3)

Answers

Answer:

The simplified form is -26x^2 + 56x -15

Step-by-step explanation:

We need to solve the expression:

3x(-2x+7)-5(x-1)(4x-3)

Multiplying the terms outside the bracket with the terms inside the bracket.

=-6x^2+21x-5(x(4x-3) -1(4x-3))

= -6x^2+21x-5(4x^2-3x-4x+3)

= -6x^2+21x-5(4x^2-7x+3)

Now multiply -5 with the terms inside the bracket

= -6x^2+21x -20x^2 +35x -15

Now, Combining the like terms:

= -6x^2 -20x^2 +21x+35x -15

Adding the like terms

= -26x^2 + 56x -15

So, the simplified form is -26x^2 + 56x -15

-26^2+56x-15 yea that should be the answer

Suppose the u.s. government put a 'special 20 percent luxury tax' on the retail price of expensive and fancy yachts in order to collect more taxes from boat owners. assume the price elasticity for these yachts is elastic at 2.50. conclusion: we can probably expect that yacht sales will go down and the government will not collect lots of new tax revenues.

Answers

True. We can probably expect that yacht sales will go down and the government will not collect lots of new tax revenues.

It is given that the US government has put a 20% luxury tax and the price elasticity for these yachts is elastic at 2.50.

Due to the new tax on luxury goods, it can be estimated that the sales of yachts go down as the maximum retail price of the yachts will become more expensive. This can mean that fewer people than before will be able to afford the yachts. Due to fewer sales, the government will not have enough tax revenues.

To learn more about price elasticity visit: https://brainly.com/question/4610585

#SPJ4


Determine if the sequence 4, 10, 19, 31,... is arithmetic. If it is, determine the first term a and the common difference d

Answers

Answer:

It is not arithmetic, so therefore there is no common difference

Step-by-step explanation:

An arithmetic sequence is a sequence where each term increases by adding/subtracting some constant k. There is no constant term in the above pattern.

Answer:

  not an arithmetic sequence

Step-by-step explanation:

An arithmetic sequence is a sequence of numbers that have a common difference. That is, the difference between any term and the previous term is constant for the sequence. Other kinds of sequences have other relationships between the differences.

Differences

The "first" differences of this sequence are ...

10-4 = 619-10 = 931 -19 = 12

The first differences are not constant. However, we notice the "second" differences are constant. These are the differences of successive first differences.

9 -6 = 312 -9 = 3 . . . . . . constant 2nd differences

Sequence type

The first differences are not constant, so this sequence is not an arithmetic sequence.

For polynomial sequences, the level of constant difference tell you the degree of the polynomial describing the sequence. This sequence has constant 2nd-level differences, so can be described by a 2nd degree (quadratic) polynomial:

  f(n) = 1.5n² +1.5n +1 . . . . . a quadratic sequence

__

Additional comment

Sequences that are exponential have differences that have a common ratio. That ratio is the same at every level. It is the base of the exponential function.

What is the multiplicative rate of change of the function? two-thirds three-fourths four-thirds three-halves

Answers

The multiplicative rate of change is 2/3

How to determine the multiplicative rate of change?

The complete question is in the image

The multiplicative rate of change is then calculated as:

r = y2/y1

This gives

r = 4/6

Simplify

r = 2/3

Hence, the multiplicative rate of change is 2/3

Read more about rate of change at:

https://brainly.com/question/4319809

#SPJ1

Answer:

b.

Step-by-step explanation:

Is (−1, −1) a solution to the following system of inequalities? Show work to verify algebraically. Then, provide an explanation below.

Answers

Yes this is a solution.

A way you can figure this out is by plugging in (-1, -1) to their corresponding values in both equations. The first -1 would replace all x values. The second -1 would replace all Y values.

After simplifying the first equation gives
-1 >= -4 + 3
-1 >= -1 TRUE

The second equations gives

-1 - (-2) >= 1
1 >= 1 TRUE

A mapping diagram showing a relation, using arrows, between input and output for the following ordered pairs: (negative 3, negative 9), (2, negative 6), (negative 5, 4), (1, 2), (6, 0).
What is the domain of the function shown in the mapping?

Answers

Answer:

{-3, 2, -5, 1, 6}

Step-by-step explanation:

The domain is the set of input values.

Which function represents exponential growth?
O f(x) = 3x
O f(x) = x³
Of(x) = x + 3
O f(x) = 3x

Answers

Answer:

b step by step explanation

The function that represents exponential growth is:

f(x) = x³

What is Exponential Growth?

An exponential function's curve is created by a pattern of data called exponential growth, which exhibits higher increases over time.

y = a(1+r)ˣ,

where a is the initial population and r is the rate in decimals and x is the time period.

Exponential growth is a phenomenon that occurs when the rate of change of a quantity is proportional to its current value. In other words, as the value of the quantity increases, the rate at which it increases also increases. This can be represented mathematically using an exponential function of the form f(x) = abˣ, where a is the initial value, b is the growth factor, and x is the time or number of periods.

The function that represents exponential growth is:

f(x) = x³

To learn more about the exponential growth;

https://brainly.com/question/12490064

#SPJ7

Someone please help with this

Answers

2sin(x) - 1 =0

first, simplify and isolate sin(x)

2sin(x) - 1 = 0

2sin(x) = 1

sin(x) = 1/2

Then, do sin inverse of 1/2 which is 30 degrees

150 degrees also works because when you set 30 degrees as the reference angle in quadrant 2, you get 150 degrees. 180-30 = 150

C is the answer

helppppp will give brainliest

Determine whether the relationship is an inverse variation or not. Explain.

a The product xy is not constant, so the relationship is an inverse variation.

b The product xy is constant, so the relationship is not an inverse variation.

c The product xy is constant, so the relationship is an inverse variation.

d The product xy is not constant, so the relationship is not an inverse variation

Answers

Answer:

The product xy is constant, so the relationship is an inverse variation.

Step-by-step explanation:

Inverse variation is the mathematical relationship between two variables which can be expressed by an equation in which the product of two variables is equal to a constant.

inverse variation is the relationships between variables that are represented in the form of y = k/x, where x and y are two variables and k is the constant value. It states if the value of one quantity increases, then the value of the other quantity decreases.

Here, the product xy is constant and gives the product as 960.

The relationship is inverse relationship.

Learn more about inverse  variation - https://brainly.com/question/2798700

#SPJ10

The question is in the photo! Please help

Answers

the answer is the second one because in order to create a perpendicular line, the slope of the new equation has to be the opposite reciprocal of the previous slope. An opposite is simply switching signs (Ex: 5 become -5), and a reciprocal is simply switching the numerator and denominator. (Ex: 2 becomes 1/2). so the opposite reciprocal would be switching the sign then flipping the numerator and denominator.
the y intercept does not matter with linear equations
the answer is the second one.

I need a written answer for all three questions please.

Answers

The values of the trigonometry ratios are:

cos α = - 5/13 and cot α = 5/12cot α = -5/12 and sec α = 13/5

How to solve the trigonometry ratios?

1: sin α = -12/13 and tan α > 0, find cos α and cot α

Because tan α > 0, then it means that cos α and sin α are negative

So, we have:

sin²α + cos²α = 1

Substitute sin α = -12/13

(-12/13)² + cos²α = 1

This gives

cos²α = 1 - (-12/13)²

Evaluate the squares

cos²α = 1 - 144/169

Evaluate the difference

cos²α = 25/169

Take the square root of both sides

cos α = - 5/13

The cotangent ratio is represented as:

cot α = cos α/sin α

This gives

cot α = (-5/13)/(-12/13)

Evaluate

cot α = 5/12

Hence, cos α = - 5/13 and cot α = 5/12

2: tan α = -12/5 for α in quadrant IV, find sec α and cot α

Because α is in quadrant IV, then it means that sec α is positive

cot α = 1/tan α

This gives

cot α = 1/(-12/5)

Evaluate

cot α = -5/12

Also, we have:

sec²α = 1 + tan²α

Substitute tan α = -12/5

sec²α = 1 + (-12/5)²

Evaluate the squares

sec²α = 1 + 144/25

Evaluate the sum

sec²α = 169/25

Take the square root of both sides

sec α = 13/5

Hence, cot α = -5/12 and sec α = 13/5

Read more about trigonometry ratios at:

https://brainly.com/question/11967894

#SPJ1

Hi can someone assist me with this question and help me solve it?

Answers

Answer:

40°

Step-by-step explanation:

Given l \\ m,

m∠13 = m∠7 (alternate interior angles)

m∠13+m∠15 = 180° (Sum of angles in a straight line)

[tex]6x+4+(14x-4)=180\\6x+4+14x-4=180\\20x=180\\x=\frac{180}{20} \\=9\\\\[/tex]

Substitute x to find m∠13

m∠13= 6x+4 = 6(9)+4 = 36 + 4 = 40°

Line / has a slope of. The line through which of the following pair of points is
perpendicular to /?
A. (4, 6), (2, 2) B. (8,8), (2, 4)
C. (8,2), (2,4)
D. (6,2), (2, 4)

Answers

The slope of a line.

Let [tex]A(x_A,\ y_A),\ B(x_B,\ y_B)[/tex]. Then a slope of the line AB represent the formula:

[tex]m=\dfrac{y_B-y_A}{x_B-x_A}[/tex]

Substitute the coordinate od the points to the formula of a slope.

[tex]A.\\(4,\ 6),\ (2,\ 2)\\\\m=\dfrac{2-6}{2-4}=\dfrac{-4}{-2}\\\\\huge\boxed{m=2}[/tex]

[tex]B.\\(8,\ 8),\ (2,\ 4)\\\\m=\dfrac{4-8}{2-8}=\dfrac{-4}{-6}\\\\\huge\boxed{m=\dfrac{2}{3}}[/tex]

[tex]C.\\(8,\ 2),\ (2,\ 4)\\\\m=\dfrac{4-2}{2-8}=\dfrac{2}{-6}\\\\\huge\boxed{m=-\dfrac{1}{3}}[/tex]

[tex]D.\\(6,\ 2),\ (2,\ 4)\\\\m=\dfrac{4-2}{2-6}=\dfrac{2}{-4}\\\\\huge\boxed{m=-\dfrac{1}{2}}[/tex]

How do you solve this?

Answers

Step-by-step explanation:

to "solve" this I need an equation.

this whole expression must be equal to something.

without that I can only try to simplify the expression.

remember that

a/b / c/d = ad / bc

so, here we have

3a/(((a²/x) - 1)(a/x - 1)) = 3a/(a³/x² - a²/x - a/x + 1) =

= 3a/(a³/x² - a²/x - a/x - a/a) =

= 3a/((a²/x² - a/x - 1/x - 1/a)×a) = 3/(a²/x² - a/x - 1/x - 1/a)

Can someone take a look at this image and answer please!

Answers

The answers to the following question is

1) KL = 10

2) TE= 15 cm

3) angle PUT= 13

4) angle SQP = 21

What is similarity in triangle?

Two triangles are similar if they have the same ratio of corresponding sides and equal pair of corresponding angles. If two or more figures have the same shape, but their sizes are different, then such objects are called similar figures.

1) Using similarity property in given triangle

SR/ KI= SJ / JK

AS, JK= 2 SJ

SR/ KI= SJ / 2 SJ

5/ KI =  1/2

KL= 10

2) As, the diagonal of parallelogram bisect equally each other then

VE = TE

AS, VE = 15

So, TE= 15 cm

3) As, PU is the bisector of angle SUT

angle PUT= 1/2 (angle SUT)

PUT = 1/2 (26)

angle PUT= 13

4) As PQ is the bisector SQR

angle PQR= angle SQP = 21

Learn more about similarity of triangles

https://brainly.com/question/25882965

#SPJ1

Se calcula que el 2013 habia 19,000 árboles en la ciudad. el año 2014.para prevenir caídas de árboles viejos, se corto el 2% y el 2015 se quemó en un incendio el 5% de lo que quedaba
A) En que porcentaje disminuyó el número de árboles de 2013 a 2015?
B) Cuantos árboles quedan en esta ciudad a fines del 2015?

Answers

Answer:

yyjyidsksjsjsosiddhdoehsudidhdhsjidjsjsjejsksksksjsjsjdd


Parallelogram EASY is drawn with diagonal ES. The measure of Angle AES is 40 degrees and the measure of Angle Y is 110
degrees.
Find the measure of Angle A, Angle YEA, Angle ESA, and Angle ESY.

Answers

Answer:

Step-by-step explanation:

Givens

Parallelogram EASY with diagonal ES

<AES = 40

<Y = 110

Solution

<AES = 40               GIVEN

<ESY = 40                     Z THEOREM OF A PRALLELOGRAM

<A = <Y                         PROPERTY OF A PARALLELOGRAM

<A = <110                      <A = 110 BECAUSE <Y = 110

<YES = 180 - <Y           -<ESY EVERY TRIANGLE HAS 180o

<YES = 180 - 110 - 40    SIMPLIFY

<YES = 30

<YEA = <YES + <AES    WE KNOW BOTH ANGLES ON THE RIGHT.

<YEA = 30 + 40             COMBINE

<YEA = 70

You could do <YEA by noting that consecutive angles of a Parallelogram equal 180

Need help to understand how to do this.

Answers

Answer: -1(x+2)^2+10

Step-by-step explanation:


1. The coefficient of -x^2 is just the number in front, which in this case would be a = -1 So now the function is -1(x^2+4x)+6 since negative one has been factored out of the first two terms.

2. Half of the coefficient of x would be 2 since x has a coefficient of 4, and half pf 4 is 2. Squared, it is 4. That will be added inside the bracket. Subtracted on the outside is the same equation but multiplied by a, which we found out was -1 in the beginning. So, the function becomes -1(x^2+4x+4)+6-(-4).

3. Factor the inside equation to find that it’ll become (x+2)^2, and simplify the outside to get 10. The function will now be;


f(x)=-1(x+2)^2+10


Hope that made sense and make sure to check just in case I did the math wrong :]

Which are true about the pyramid? Check all that apply.
The base of the pyramid is a square with four sides, each measuring 756 feet.
The slant height of the pyramid is 612 feet.
The slant height of the pyramid is 756 feet.
All four triangular faces are congruent.
All four triangular faces are not congruent.
The pyramid has four lateral faces.

Answers

The statements that are true about the pyramid are

The base of the pyramid is a square with four sides, each measuring 756 feet.The slant height of the pyramid is 612 feet.All four triangular faces are congruent.The pyramid has four lateral faces.

Properties of a Square pyramid

From the question, we are to determine the statements that are true about the pyramid

The diagram shows a square pyramid,

The length of a side of its base is 756 ft

and

The height / altitude of a triangular face is 612 ft

Since the pyramid is a square pyramid,

Then, the base must be a square with each side measuring 756 feet

∴ The first statement is true

Since the height of a triangular face is 612 ft,

Then,

∴ The slant height of the pyramid is 612 feet

NOTE: Slant height of a pyramid is the altitude of the triangle comprising a lateral face

Thus, the second statement is true

Since the triangular faces each have a base of 756ft and a height of 612 ft,

Then,

All four triangular faces are congruent

∴ The fourth statement is true

NOTE: Lateral sides of a solid are the faces on the sides of the shape

The pyramid has four lateral faces which are each a triangle

∴ The last statement is true

Hence, the statements that are true about the pyramid are

The base of the pyramid is a square with four sides, each measuring 756 feet.The slant height of the pyramid is 612 feet.All four triangular faces are congruent.The pyramid has four lateral faces.

Learn more on Pyramid here: https://brainly.com/question/10042135

#SPJ1

Pls help ill give brainiest

Answers

The last two need to be flipped

I am lazy to type so here look at the picture and there is your answer

Q2 - Using fractions
Jarvis works in a garage for $7 an hour.
If he works on Saturday he is paid time and a quarter.
If he works on Sunday he is paid time and three quarters.
Last weekend Jarvis worked for four hours on Saturday and four hours on Sunday.
How much was Jarvis paid last weekend altogether?

Answers

Total Earned Saturday: $45

Total Earned Sunday: $63

Total: $108

Time and a quarter means that you divide his normal rate by 4 and add that to his normal rate.

9/4 = 2.25     2.25 + 9 = 11.25

Jarvis makes $11.25 an hour on Saturdays.

Since he worked 4 hours, you multiply,

11.25 x 4 = 45.

What is the normal rate?

Time and three quarters mean that you multiply his normal rate by 3/4 and add that to his normal rate

9x0.75 = 6.75     6.75 + 9 = 15.75

Jarvis makes $15.75 an hour on Sundays. Since he worked 4 hours, you multiply 15.75 x 4 = 63.

To get the total, you add his earnings on Saturday and earnings on Sunday

45 + 63 = 108.

To learn more about the using fractions visit:

https://brainly.com/question/78672

#SPJ1

Describe the
translation.
y = (x - 5)² +5 →
y = (x −0)² +0
A.T<5,-5>
OB.T<-5,-5>
OC.T<-5,5>
OD. T<5,5>

Answers

The translation of the parabola from y = (x – 5)² + 5 to y = (x − 0)² + 0 will be (-5, -5). Then the correct option is B.

What is the parabola?

It's the locus of a moving point that keeps the same distance between a stationary point and a specified line. The focus is a non-movable point, while the directrix is a non-movable line.

The equation of a quadratic function, of vertex (h, k), is given by:

y = a(x – h)² + k

where a is the leading coefficient.

The translation of the parabola is given below.

y = (x – 5)² + 5 → y = (x − 0)² + 0

Then the translation will be (-5, -5).

Then the correct option is B.

More about the parabola link is given below.

https://brainly.com/question/8495504

#SPJ1

Given the matrices A and B shown below, find -A + 1/3B

Answers

Step-by-step explanation:

1. first multiply -1 times all elements of matrix A

2. then multiply 1/3 by all elements of matrix B

3. then add each corresponding entries to get the result.

from step 1. matrix A will be

-4 -2. -1. -3

-2. 0. 1. -3

step 2. matrix B will be

3. -1. -2. -4

3. -10. 10. -1

add each corresponding elements to get

-1. -3. -3. -7

1. -10 11. -4

3 x 2/5 converted into a mixed number

Answers

Answer:

[tex]1 \frac{1}{5} [/tex]

Step-by-step explanation:

[tex]3 \times \frac{2}{5} \\ \frac{3}{1} \times \frac{2}{5} = \frac{6}{5} \\ \frac{6}{5} = 1 \frac{1}{5} [/tex]

assume that y varies inversely with x. If y= 1.6 when x= 0.5, find x when y= 3.2

Answers

Answer:

x = 0.25

Step-by-step explanation:

If y varies inversely with x, then:

[tex]y \propto \dfrac{1}{x} \implies y=\dfrac{k}{x} \quad \textsf{(for some constant k)}[/tex]

Given:

y = 1.6 when x = 0.5

Substitute the given values into the found equation and solve for k:

[tex]\implies 1.6=\dfrac{k}{0.5}[/tex]

[tex]\implies k=1.6(0.5)[/tex]

[tex]\implies k=0.8[/tex]

Therefore:

[tex]y=\dfrac{0.8}{x}[/tex]

To find the value of x when y = 3.2, substitute y = 3.2 into the found equation and solve for x:

[tex]\implies 3.2=\dfrac{0.8}{x}[/tex]

[tex]\implies x=\dfrac{0.8}{3.2}[/tex]

[tex]\implies x=0.25[/tex]

The set of life spans of an appliance is normally distributed with a mean mu = 48 months and a standard deviation sigma = 8 months. what is the z-score of an appliance that stopped working at 64 months?

Answers

The z-score of an appliance that stopped working at 64 months is 2.

What is mean?

The mean of observations is equal to the ratio of sum of all the observations and the number of observations.

Given, the set of life spans of an appliance is normally distributed with a mean mu = 48 months and a standard deviation sigma = 8 months

The z-score is then calculated as

z = x - μ /σ

For an appliance that stopped working at 64 months, we have

x = 64

On substituting the values, we get

z = 64-48/8

z = 2

Hence, the z-score of an appliance that stopped working at 64 months is 2

Learn more about mean.

https://brainly.com/question/11822836

#SPJ1

Answer:

D on edge

Step-by-step explanation:

2

Peter uses the equation y = StartFraction 13 over 4 EndFraction x to model the number of miles that he has walked in x hours. Which statement is true about the proportional relationship that is modeled by Peter’s equation?

Answers

Regarding the proportional connection that Peter's equation simulates, claim C is accurate. Peter moves along at a 13/4 mph walking pace.

What is the equation?

An equation is a mathematical statement that uses an equal sign to join two algebraic expressions having the same value.

The complete question is

"Peter uses the equation Y=13/4x to model the number of miles that he has walked in x hours. Which statement is true about the proportional relationship that is modeled by the peat there's the equation?

A:Peter walks a rate of 4/13 miles per hour.

B: Peter walks at a  rate of 4 miles per hour.

C: Peter walks at a rate of 13/4 miles per hour.

D: Peter walks at a rate of 13 miles per hour."

Provided equation

Y=13/4x

where y is the distance traveled and x denotes the amount of time.

According to the equation, Peter moves along at a speed of 13/4 miles per hour.

As a result, the proportional connection represented by Peter's equation is valid as stated in assertion C.

To learn more, about equations, refer;

https://brainly.com/question/10413253

#SPJ1

A couple plans to have 4 children. The gender of each child is equally likely. Design a simulation involving 55 trials that you can use to model the genders of the children. Write your answer as number

Answers

Answer:

4x55x2

Step-by-step explanation:

4 kids 55trys 50 50 odds

Other Questions
Kinetic energy of a body is increased by by 300% what is percentage increased in momentum Does this graph represent a proportional relationship? Explain.On a coordinate plane, a curved line has 2 curves. This whole page i need help on Select the graphs that show a proportional relationship between x and y Ilene rents a property for the entire year. During the year, Ilene reported a net loss of $15,000 from the rental. If Ilene is an active participant in the rental and her AGI is $140,000, how much of the loss can she deduct against ordinary income in the year What is the exact value of sin (- 105)? These tables of values represent continuous functions. For which functionwill the y-values be the greatest for very large values of x?y1 72 213 634 1895 567B. xy172 2073 4074 6075 807C.xy1 3252 4003 4754 5505625Y1 5652 5703 5754 5805585What is the answer?? Mr. Sinclair has diabetes and heart trouble and is generally satisfied with the care he has received under Original Medicare, but he would like to know more about Medicare Advantage Special Needs Plans (SNPs). What could you tell him Question 5 of 10Choose the symbol that correctly compares the fracti--IN1131/2 1/2A. >B. =C. what is the average daily balance for the February 1-28 billing period in the table? in what country did communists come topower in 1959 and why HJ=help me please thannk u :) Pls answer the question 15 points An Interest Only Strip holder benefits from _____ than expected prepayments, and a Principal Only Strip holder benefits from lower than expected _____. A group of people were given a personality test to determine if they were Type A or Type B. The results are shown in the table below: Male Female Type A 65 85 Type B 38 12 Compare P(Male or Type B) with P(Male | Type B). P(Male or Type B) > P(Male | Type B) P(Male or Type B) = P(Male | Type B) P(Male or Type B) < P(Male | Type B) There is not enough information. Sort the length measurements in increasing order of magnitude from smallest to largest. A prescriber orders clonidine ER (Kapvay ER) tablets for a 12-year-old child to treat which condition Which membranous structures conduct impulses through the sarcoplasm to stimulate the release of calcium?. Question 33 of 45 You may use your calculator for this question. A particle moves along a line so that at time t where 0 tn, its position is given by s(t)=-4 sint- t/2+10. What is the acceleration of the particle the 2 first time its velocity equals zero? PLEASE HELP ME I GIVE BRAINLEST List the two qualities of lifetime sports activities.low impact and contacthigh impact and contacthigh Impact and no contactlow Impact and no contactI need help with the contact part. Is it low contact or no contact?